2
$\begingroup$

Let $L = \{a^{f(m)} | m \geq 1 \}$ where $f: \mathbb{Z}^+ \rightarrow \mathbb{Z}^+$ is monotonically increasing and complies that for all $n \in \mathbb{Z}^+$ there is $m \in \mathbb{Z}^+$ such that $f(m+1) - f(m) \geq n$.

Thanks in advance,

Regards,

Alex.

  • 2
    please consult the [faq](http://cstheory.stackexchange.com/faq): cstheory is for research level questions. We have a strict policy of no homework questions.2011-08-16
  • 0
    Please read the FAQ. I am migrating the question to Math.SE.2011-08-16

2 Answers 2